Difference between revisions of "2022 AMC 10B Problems/Problem 9"
(→Solution) |
(→Solution) |
||
Line 6: | Line 6: | ||
<math> \textbf{(A)}\ 2020 \qquad\textbf{(B)}\ 2021 \qquad\textbf{(C)}\ 2022 \qquad\textbf{(D)}\ 2023 \qquad\textbf{(E)}\ 2024</math> | <math> \textbf{(A)}\ 2020 \qquad\textbf{(B)}\ 2021 \qquad\textbf{(C)}\ 2022 \qquad\textbf{(D)}\ 2023 \qquad\textbf{(E)}\ 2024</math> | ||
− | ==Solution== | + | ==Solution 1== |
Note that <math>\frac{n}{(n+1)!} = \frac{1}{n!} - \frac{1}{(n+1)!}</math>, and therefore this sum is a telescoping sum, which is equivalent to <math>1 - \frac{1}{2022!}</math>. Our answer is <math>1 + 2022 = \boxed{\textbf{(D)}\ 2023}</math>. | Note that <math>\frac{n}{(n+1)!} = \frac{1}{n!} - \frac{1}{(n+1)!}</math>, and therefore this sum is a telescoping sum, which is equivalent to <math>1 - \frac{1}{2022!}</math>. Our answer is <math>1 + 2022 = \boxed{\textbf{(D)}\ 2023}</math>. | ||
Line 12: | Line 12: | ||
~mathboy100 | ~mathboy100 | ||
− | + | ==Solution 2== | |
We have <math>(\frac{1}{2!}+\frac{2}{3!}+\dots+\frac{2021}{2022!})+\frac{1}{2022!}=(\frac{1}{2!}+\frac{2}{3!}+\dots+\frac{2020}{2021!})+\frac{1}{2021!}</math> from canceling a 2022 from <math>\frac{2021+1}{2022!}</math>. | We have <math>(\frac{1}{2!}+\frac{2}{3!}+\dots+\frac{2021}{2022!})+\frac{1}{2022!}=(\frac{1}{2!}+\frac{2}{3!}+\dots+\frac{2020}{2021!})+\frac{1}{2021!}</math> from canceling a 2022 from <math>\frac{2021+1}{2022!}</math>. | ||
This sum clearly telescopes, thus we end up with <math>(\frac{1}{2!}+\frac{2}{3!})+\frac{1}{3!}=\frac{2}{2!}=1</math>. Thus the original equation is equal to <math>1-\frac{1}{2022}</math>, and <math>1+2022=2023</math>. <math>\boxed{\textbf{(D)}\ 2023}</math>. | This sum clearly telescopes, thus we end up with <math>(\frac{1}{2!}+\frac{2}{3!})+\frac{1}{3!}=\frac{2}{2!}=1</math>. Thus the original equation is equal to <math>1-\frac{1}{2022}</math>, and <math>1+2022=2023</math>. <math>\boxed{\textbf{(D)}\ 2023}</math>. |
Revision as of 21:15, 17 November 2022
Contents
Problem
The sum can be expressed as , where and are positive integers. What is ?
Solution 1
Note that , and therefore this sum is a telescoping sum, which is equivalent to . Our answer is .
~mathboy100
Solution 2
We have from canceling a 2022 from . This sum clearly telescopes, thus we end up with . Thus the original equation is equal to , and . .
~not_slay
See Also
2022 AMC 10B (Problems • Answer Key • Resources) | ||
Preceded by Problem 8 |
Followed by Problem 10 | |
1 • 2 • 3 • 4 • 5 • 6 • 7 • 8 • 9 • 10 • 11 • 12 • 13 • 14 • 15 • 16 • 17 • 18 • 19 • 20 • 21 • 22 • 23 • 24 • 25 | ||
All AMC 10 Problems and Solutions |
The problems on this page are copyrighted by the Mathematical Association of America's American Mathematics Competitions.